MBE Civ pro Flashcards

1
Q

SMJ:

A

Jurisdiction over the kind of case

Issue: Does this court have the power to decide this kind of case?

Federal courts are courts of limited subject-matter jurisdiction.

State courts are generally courts of general subject-matter jurisdiction.

VVVVVVVVVVVVVVVVVVVVVVVVVVVV
Pleading

Basis for federal SMJ must be affirmatively pleaded in every case.

If challenged, it must be proved that there is a basis for SMJ.

VVVVVVVVVVVVVVVVVVVVVVVVVVVV

Waiver

No waiver for a lack of SMJ (parties cannot agree between themselves to litigate in a court that lacks SMJ)

Lack of SMJ cannot be waived by failing to object or by affirmative consent

VVVVVVVVVVVVVVVVVVVVVVVVVVVV

objection

Lack of SMJ can be raised by any party at any time, including by:
* The plaintiff, who chose to go to the wrong court
* The court itself
* Anyone for the first time on appeal

Exam Tip 1: For the MBE, unless you are given a state statute that says otherwise, assume state courts have subject-matter jurisdiction over any kind of case.

How well did you know this?
1
Not at all
2
3
4
5
Perfectly
2
Q

subject matter jurisdiction: diversity jurisdiction –>amount in controversy (aggregation)

A

aggregation: adding up smaller claims to exceed $75,000

  1. one plaintiff v. one defendant: plaintiff can aggregate all of her claims, regardless of whether claims are related
  2. one plaintiff v. two (or more) defendants: plaintiff may not aggregate her claims against multiple defendants
  3. two (or more) plaintiffs v. one defendant: plaintiffs may not aggregate their claims against the defendant

supplemental jurisdiction: smaller-value claims are sometimes permitted as a matter of supplemental jurisdiction

How well did you know this?
1
Not at all
2
3
4
5
Perfectly
3
Q

SMJ: Complete Diversity

A

The diversity statute requires complete diversity.

Every citizenship represented on the plaintiff’s side of the case must be different from every citizenship represented on the defendant’s side.

Does NOT mean every plaintiff must be diverse from every other plaintif for that the defendant must be diverse from every other defendant

Exception: Minimal Diversity

Exists when any plaintiff is diverse from any defendant, even if other plaintiffs and defendants overlap.

Minimal diversity is permitted in the following circumstances:

  • Federal Interpleader Act (statutory interpleader);
  • Class actions with at least 100 class members and claims worth more than $5 million (Class Action Fairness Act); and
  • Interstate mass torts if at least 75 natural persons have died in one accident and the plaintiffs and defendants are from many different states (airline crash).

Outside these limited exceptions, the general rule is that complete diversity is required.

Diversity must only be complete as between plaintiffs and defendants.

How well did you know this?
1
Not at all
2
3
4
5
Perfectly
4
Q

SMJ: Citizenship of the Parties

A

Individuals

Citizen of the state or country of domicile
* Must be a citizen of the United States and a domiciliary of the relevant state
* Domicile is permanent residence, (residence + an intent
to remain indefinitely)
For purposes of jurisdiction, there can onlyy be one legal domicile at a time

Aliens (Citizens or Subjects of a Foreign Country)

Diversity jurisdiction exists for controversies between a citizen (or citizens) of a state and a citizen (or citizens) of a foreign country.

A citizen of a foreign country who has been admitted to the U.S. as a permanent resident is treated as a citizen of the state where she is domiciled.

Diverse citizenship does not exist when a suit involves stateless persons:
(1) noncitizens present in the U.S. but not citizens of a foreign country or
(2) U.S. citizens domiciled in a foreign country.

Representative parties

Generally, the citizenship of the representative controls.

Exception: For litigation involving a decedent’s estate, the citizenship of the decedent controls.

Exception: For the legal representative (guardian) of a minor or an incompetent person, the domicile of the infant or incompetent person controls.

Class actions

Citizenship of the named or representative parties count

Class members not named may join without regard to citizenship.

Corporations

Can have several citizenships simultaneously
Citizen of:
* The state, states, or foreign countries in which it is incorporated; and
* The state or country where it has its principal place of business
(where its executive offices are located)

Remember, when a corporation is a party, you still need COMPLETE diversity between plaintiffs and defendants, so you must consider EVERY state where the corporation has citizenship.

How well did you know this?
1
Not at all
2
3
4
5
Perfectly
5
Q

SMJ: Devices to Create or Destroy Diversity

A

Actions that create or destroy diversity are permitted so long as they are not “shams” or fraudulent.

  • Moving: permitted, even if it is done with the purpose of affecting diversity, so long as the change in domicile is genuine, not a sham
  • Assignment of a claim: permitted so long as the assignment is real (for value), complete, and not collusive
  • Partial assignment of a claim for the purpose of debt collection does NOT affect citizenship if the assignor retains an interest in the claim.
How well did you know this?
1
Not at all
2
3
4
5
Perfectly
6
Q

removal

A

Removal moves case from state court to federal court.

Transfer moves case from one federal court to another federal court.

No procedure for removal of a case from federal court to state court.
* Federal court can abstain from hearing the case (in very narrow circumstances).

VVVVVVVVVVVVVVVVVV

general rule for removal

Removal is proper ONLY if the case could have been brought originally in federal court.

ONLY defendants may remove.
* ALL defendants must consent to removal within 30 days of service (does not have to be at the same time in a single notice, all must consent within 30 days of service upon plaintiff)

ASK: If the plaintiff had chosen to sue in federal court in the first place, would the federal court have had subject matter jurisdiction?

VVVVVVVVVVVVVVVVVV

Removal of Federal Question Cases

Well-pleaded complaint rule: Federal question jurisdiction on whether the federal question appears on the face of the well-pleaded complaint.

Removal does not change that rule: If the well-pleaded complaint discloses that the plaintiff’s claim is based on federal law, then the defendant may remove to federal court.
* If the plaintiff’s claim is based on state law, defendant cannot remove to federal court, even if the defendant has raised a federal defense.

In cases of removal based on federal question jurisdiction, only those defendants against whom the federal claim is asserted must join in or consent to the removal

VVVVVVVVVVVVVVVVVV

removal of diversity cases

Removal based on diversity jurisdiction is proper only if:
1. There is complete diversity;
2. The amount in controversy exceeds $75,000; and
3. The action is brought in a state of which no defendant is a citizen

One-year limit on removal: Must remove within one year of the commencement of the action in state court, unless the plaintiff acted in bad faith to make the case non- removable.

VVVVVVVVVVVVVVVVVVV

removal procedures

A notice of removal is filed in the federal court, with a copy to the state court.

Once a defendant files a notice of removal, the case is automatically transferred to federal court.

VVVVVVVVVVVVVVVVVVV

improper removal

Plaintiff can file in federal court a petition for remand, and it is on that petition that the federal court will hold a hearing.
* if removal is proper, remand is denied, and case stays in fed court
* if removal is improper, remand is granted, and case goes back to state court

How well did you know this?
1
Not at all
2
3
4
5
Perfectly
7
Q

subject matter jurisdiction: supplemental jurisdiction

A

general rule: allows a federal court with SMJ over a case to hear additional claims over which the court would not independently have jurisdiction if ALL the claims constitute the same case or controversy

  • same case or controversy: arising out of the same common nucleus of operative fact; all the claims arise out of the same transaction/occurence
  • supplemental jurisdiction allows, but does not require, the court to expand its jurisdiction; decision is within discretion of trial court and typically made on practical grounds
How well did you know this?
1
Not at all
2
3
4
5
Perfectly
8
Q

supplemental jurisdiction: Federal Question Jurisdiction (“Arising Under”)

A

Plaintiff’s claim, which arises under Constitution, federal law, or treaty

Well pleaded complaint

Plaintiff’s claim must be based on federal law.

Look at the face of the plaintiff’s complaint.

Only the plaintiff’s claims count for determining federal question jurisdiction; the existence of a federal defense does not matter.

Plaintiff does not need to anticipate or plead a response to a defense.

A federal defense does not count for federal question jurisdiction, even if it is important.

a federal question claim is an “Anchor” claim and forms basis for federal court’s SMJ over a case, so if there is a state law claim which does not qualify on its own for federal SMJ if it arises out of same common nucleus of operative fact, court may exercise supplemental jurisdiction over state law claim.

this is called “pendant party jurisdiction”

How well did you know this?
1
Not at all
2
3
4
5
Perfectly
9
Q

SMJ: Diversity

A

cases between citizens of different state, or citizens of a state and foreign counrt, if the amount in controvery exceeds 75k (exclusive of interests and costs; any good faith allegation will suffice)

If you sue someone’s insurance, the insurance is a corporation; they are a citizen of:
* the state of its incorporation
* the state of its principal place of business.
* the insured’s state when the insurer is sued in a direct action.

Exceptions

probate and domestic relations actions cannot be brought in federal court under diversity jurisdiction even if the normal requirements are satisfied.

time for diversity determination

Diversity must exist when the complaint is filed

Does NOT matter that diversity did not exist when the cause of action arose.

Does NOT matter that diversity no longer exists when the case comes to trial

VVVVVVVVVVVVVVVVV

state law claims must be part of “Same case or controversy” (common nucleus of operative fact)

statute bars supplemental jurisdiction in diversity cases over:
* Claims by plaintiffs against persons made party under Rules 14, 19, 20, or 24;
* Claims by parties seeking to intervene as plaintiffs under Rule 24; or
* Claims by parties proposed to be joined as plaintiffs under Rule 19.

If such claims are to be part of a diversity case, they must independently satisfy the complete diversity and AIC requirements.

statutory nuances

Supplemental claims by plaintiffs are allowed if they satisfy the common nucleus of operative fact test.

counterclaims

A compulsory counterclaim is one that arises out of the same transaction or occurrence as the other party’s claim.

A permissive counterclaim is one that does NOT arise out of the same transaction or occurrence as the main claim.

cross-claims

Claim by a co-party (plaintiff against another plaintiff; defendant vs defendant)

Relatedness Requirement: Must arise out of the transaction or
occurrence that is the subject matter of the action or of a counterclaim.

To qualify for supplemental jurisdiction, a cross-claim:
* Must be related to a claim over which the court has subject-matter jurisdiction (i.e., the “anchoring” claim); and
* Must not be asserted by a plaintiff against a person made party under Rules 14, 19, 20, or 24.

VVVVVVVVVVVVVVVVV

multiple plaintiffs, single defendant (permissive joinders and class action)

If the claim of one diverse plaintiff against a single defendant satisfies the jurisdictional amount, other diverse plaintiffs who have related claims against the defendant can also be heard even if their claims do NOT satisfy the jurisdictional AIC minimum.

Supplemental jurisdiction is available on the same logic when multiple plaintiffs do not form a class, but choose to join together in the same action. If one plaintiff has a claim worth $75K+, the others can raise claims arising from the same transaction regardless of amount.

How well did you know this?
1
Not at all
2
3
4
5
Perfectly
10
Q

personal jurisdiction (PJ):

A

power of the court to decide rights and liabilities of this defendant

Personal jurisdiction is an issue in federal and state court, and rules are generally the same in federal and state court.
* Default federal rule—use the long-arm statute of the state in which the federal court sits (chief means of asserting PJ over out-of-state defendants).

Must always ask two questions:
1. Has the basis for exercising PJ over an out-of-state defendant been
authorized by statute or by rule of court?
2. Is the particular basis for exercising personal jurisdiction permitted by the due process Clause of the US Constitution?

VVVVVVVVVVVVVVVVV

waiver

Unlike subject-matter jurisdiction, and objection to a lack of personal jurisdiction can be waived.

voluntarily litigating on merits waives any objection to lack of PJ (a general appearance)
* A defendant waives any objection to personal jurisdiction (consents to the court’s jurisdiction) by substantial participation on the merits before raising that objection.
* this includes almost anything other than challenging jurisdiction.

federal rules: lack of PJ must be raised at the first opportunity, or it is waived
* If the defendant chooses to file a pre-answer motion to dismiss she must make the objection then or waive it, OR
* if the defendant does not file a pre-answer motion, the objection to personal jurisdiction must be raised in the answer.

VVVVVVVVVVVVVVVVV

types of personal jurisdiction

in-personam: against the person
in-rem: against the thing
quasi in-rem: “sort of” against the thing

VVVVVVVVVVVVVVVVVVVV

in personam jurisdiction

(1) constitutional aspect

due process requires minimum contacts between defendant and forum state

there must be sufficient contacts between the defendant and the forum so that it would be consistent with traditional notions of fair play and substantial justice to sue the defendant there

in assessing minimum contacts, courts look to see whether defendant has purposefully associated himself with the forum in some way
* Contacts between the forum and the plaintiff do NOT suffice.
* There must be minimum contacts between the forum and the defendant
* Those contacts must be a result of the defendant’s own purposeful actions

(2) general and specific in personam jurisdiction
* federal courts follow the personal jurisdiction law of the states in which they sit
* there is some variation state to state, but not much

general in personam jurisdiction: The plaintiff can assert any claim whatsoever, even if it is unrelated to the defendant’s contacts with the forum state.

specific in personam jurisdiction:
* Plaintiff’s claims against the defendant must arise out of—or directly relate to—the defendant’s contacts with the forum.
* State long-arm statutes authorize specific jurisdiction based on some minor contacts.

bases for general in personam jurisdiction

physical presence within the state: Service of process on the defendant while she is physically present in the state

exceptions: If a person was in the state only to answer a summons or Persons brought to the state by brought there by force or fraud

If person is in the state knowingly and voluntarily, then the person can be served there.

domicile
Domicile is independent of physical present.

consent
Expressly (“Yes, I’d be delighted”) or implicitly (by not raising the issue)

Can be given by appointment of an agent for receiving of process within
the state, or by contract.

corporations
Corporations can consent to personal jurisdiction just like individuals, but the rules regarding physical presence are a little different.

Merely “doing business” in a state is insufficient to support general jurisdiction.

General in personam jurisdiction applies only to corporations that are essentially
“at home” in the forum state.
* The state of incorporation; and
* The state where the corporation has its principal place of business.

  • where court always has PJ:
    1. Where you reside in that state- state has PJ
    2. If you are physically present in state and served there
    3. when you consent to be sued in jurisdiction (like in k)

to get PJ on out-of-state party:
1. when person has sufficient minimum contacts like frieds, family, etc
2. if it is a business conducting business in state look for whether the business is systemic, continuous, purposeful or if they purposefully availed themselves of law of statwe (filing there as business bc of tax rate)

if one has sufficient minimum contacts it will not violate their due process rights (fair) and defendant should have reasonably anticipated litigation of case

How well did you know this?
1
Not at all
2
3
4
5
Perfectly
11
Q

specific in personam jurisdiction

A

constitutional test: defendants must have sufficient minimum contacts with the forum state such that exercise of jurisdiction is fair

statutes

every state has a long-arm statute ; give courts in personam jurisdiction over out-of-state defendants but onyl for the particular transactions involving that state
* Some state statutes are very broad, “the long-arm statute extends as far as the Constitution allows”
* Others list specific activities that give rise to jurisdiction (see below)

common bases for specific in personam jurisdiction

any acts or ommission in the state causing injury to a person or property here or elsewhere

an act or omission inside the state causing injury to person/property within the state, provided that defendant conducted activities here/introduced goods into flow of commerce

any claim arising out of contract to perform service in state/pay someone in state to perform services elsewhere

any claim regarding local property

any action against officer/director of domestic corporation for acts taken in that capacity

any contract for insurance where plaintiff was resident of state when claim arose, or event giving rise to claim occurred in the state

federal exceptions

generally, fed cts follow state long-arm statutes of states in which they sit

the exceptions are federal rules that extend in personam jurisdiction of federal courts beyond state boundaries.

federal interpleader act:
* Authorizes nation-wide service of process
* Service anywhere in the United States establishes personal jurisdiction
* For a federal court, the relevant jurisdiction is the U.S. as a whole, so congress can authorize nationwide service of process as means of asserting personal jurisdiction nationwide

bulge provision of federal rules:
Allows service anywhere within 100 miles of the federal courthouse, even if in another state, in two situations:
1. For impleading third-party defendants under Rule 14; and
2. For joining necessary parties under Rule 19.

unusual provision:

  • Use: Defendant not subject to personal jurisdiction in any state court
  • Rule 4(k)(2): applies only when plaintiff is suing under federal law
  • When the plaintiff is suing under federal law and no state has jurisdiction over this defendant, the federal court can exercise jurisdiction so long has the defendant has sufficient contacts with the U.S. as a whole.
  • Used rarely; most relevant with foreign defendants with
    no particular contact with a particular state
How well did you know this?
1
Not at all
2
3
4
5
Perfectly
12
Q

in rem and quasi in rem jurisdiction

A

in rem jurisdiction

suit against a thing (the res) vs suit against a person

utility: suit can be brought against a piece of property so long as
* property is located in state where you are suing
* suit will settle everyone’s claims to the property

the res: may be any property real (land) or personal (bank account)

essential: this state’s courts have power over the property (property must be located within); this is type of specific jurisdiction

quasi-in-rem jurisdiction

suit to adjudicate claim to property of a specific defendant; the subject matter of the suit may/may not be related to property

historically, possible to use quasi-in rem jurisdiction as way to get to defendant’s assets

present rule:
The SC has held quasi-in-rem action subject to same minimum contacts test as applied to personam jurisdiction
* When the property is the subject of the suit, it will constitute an important “minimum contact” between the defendant and the forum, and jurisdiction will probably be permitted as a matter of specific in personam jurisdiction.
* When the property is not the subject of the suit, the ownership of property in the state is not nearly enough to establish general jurisdiction over the defendant

How well did you know this?
1
Not at all
2
3
4
5
Perfectly
13
Q

service of process

A

Any defendant (individual or organization) receives proper service of process when someone who is at least 18 years old and not a party to the suit serves the defendant with a summons and complaint.

generally means service of summons and a copy of the complaint on the defendant
* formally gives court jurisdiction over the case
* gives defendant notice of action
* SERVICE MUST BE MADE WITHIN 90 DAYS AFTER COMPLAINT IS FILED (unless plaintiff shows good cause as to why service was untimely, court may extend time– like plaintiff ill or injured)

personal service: used to assert in personam jurisdiction
* Should also be used for in rem and quasi-in-rem actions when the identity of an interested party is known
* Service by publication is only appropriate for in rem actions when that is the best that the serving party can do.
* Requirement: statutory authorization and constitutional validity (constitutional test: notice must be reasonable under circumstances)

Fed rules authorize service in accordance with state law where court sits OR where service is made
* default rule is that service in federal court is okay as long as it follows the law of the state in which the federal court is based.

federal rules for service ALWAYS allowed:
1. In-hand personal delivery;
2. Leaving the summons at the defendant’s dwelling or usual place
of abode with a person of suitable age and discretion;
3. Delivery of the summons to an authorized agent; and
4. For persons in foreign countries, service can be made by
registered mail, return receipt requested

special rules for service of process
* infant: service on the infant AND on the guardian
* Adjudicated incompetent: Service must be made on the incompetent AND her guardian
* Partnership: Service on a general partner, an attorney in fact, or an authorized agent
* corporation: Service on an officer, director, managing agent, or on an agent appointed for receiving service of process (corporations doing business in the state must have an agent)
* nonresident motorists: Claims arising out of in-state accidents; Generally, service can be made on a state official who forwards a copy to the out-of-
state defendant.

service of process for in rem and quasi in rem actions
* you must make a diligent effort to locate all claimants to the property (res) and serve them personally
* if the claimant cannot be located, then notice by publication is permitted

How well did you know this?
1
Not at all
2
3
4
5
Perfectly
14
Q

Service of process: waiver of service

A

Rule 4: Defendants have a duty to avoid unnecessary expenses of serving the summons by waiving service.

Plaintiff can notify defendant that an action has been commenced and request that defendant waive service of the summons.
* plaintiff can do this by 1st class mail
* defendant has 30 days to respond, or 60 days if the defendant is outside US

people waive because rules incentivize defendant by extending the deadline to file answer
* usual deadline to file an answer is 21 days after the service of process
* defendant agrees to waive service, then defendant gets 60 days to answer and 90 days if outside US

requirements for waiver
* in writing
* addressed to defendant or authorized agent
* names court where complaint filed
* provides copy of complaint, two waiver forms, and prepaid means to return form
* informs defendant about consequences of waiving and not waiving
* lists date request sent

benefit: defendant’s time to file answer extended to 60 days (90 if abroad) from date sent

consequences of waiver
* defendant pays expenses of future service of process
* defendant pays reasonable expenses of any motion required to collect service expenses (atty’s fees)

How well did you know this?
1
Not at all
2
3
4
5
Perfectly
15
Q

service of process: injunction notice issues

A

Injunction: a form of relief that requires a defendant to do something (i.e., a mandatory injunction) or prohibits a defendant from doing something (i.e., a prohibitory injunction).

After the court has fully heard the case, it can issue a permanent injunction.

VVVVVVVVVVVVV

ways to stop behaviors:

Preliminary injunction: Relief that can be issued prior to a full hearing on the
merits. Requires:
* notice to the defendant and
* hearing on whether the injunction should be granted

a federal court may grant a preliminary injunction only when it is established that:
* the movant is likely to succeed on the merits
* the movant is likely to suffer irreparable harm (injury that CANNOT be compensated by monetary damages) in the absence of relief
* the balance of equities is in the movant’s favor (the harm to the movant absent an injunction outweighs the harm an injunction would cause to the nonmovant(s)) and
* the injunction is in the best interests of the public—e.g., enforcement of contractual rights and obligations protects the freedom to contract.

VVVVVVVVVVVVV

temporary restraining order: used to preserve the status quo until the court can make a decision on the preliminary injunction.
* Temporary measure
* Cannot last more than 14 days, unless the other party agrees or the court finds
consents for an extension for “a like period.”
* A TRO can be issued wihtout notice to another party;
* Moving party has to establish, under oath, that there will be immediate and irreparable harm if there is a delay
* Moving party’s attorney has to accomplish notice in writing any efforts that have been made to give notice and the reason why notice is not required.
* If TRO is issued without notice, then the other party can make a motion to dissolve TRO
* TROs are not appealable, unless they have the effect of a preliminary injunction

How well did you know this?
1
Not at all
2
3
4
5
Perfectly
16
Q

venue

A

Where among the courts in this judicial system is the appropriate court to hear the case
* federal system: whether this venue
* it is the defendant’s responsibility to object if venue is improper.
* Federal law—claim of improper venue must be made at the first hearing or it is waived.
* Either a pre-answer motion to dismiss if defendant chooses to file one, or the defendant’s answer, if no pre-answer motion is made

VVVVVVVVVVVVVVVVVV

federal rule

Federal venue concerns which district should hear a case.

Federal venue is proper in a district:

  1. Where any defendant resides, as long as ALL defendants reside in the same state; or
  2. Where the claim arose (Where a “substantial part of the events or omissions” on which the claim is based occurred or where a “substantial part of the property” that is the subject of the action is located.)
  3. If neither of the above (rare), any district where the defendant is subject to personal jurisdiction has residence

special venue rules apply in certain situations, including when a defendant is a nonresident of the U.S.
* In this situation, venue is proper in any judicial district—including a district in the state where the Plaintiff resides.

residence:
* Individuals: residence means domicile
* business entities: any business entity (capacity to sue or be sued) resides in every district in which personal jurisdiction exists

special provisions:
For a case begun in state court and removed to federal court, venue is automatically proper in the federal district where the state court sits, even if that district would not have been proper originally.

transfer (one federal court to another):
General rule: transfer is only to a district with proper venue
* exception: Transfer to a district without proper venue may occur when ALL parties agree.

a case brought in a district with proper venue may be transferred for convenience to another district with proper venue. (if interest of justice requires)

a case brought to a district without proper venue may be
1. dismissed or
2. transferred to a district with proper venue

VVVVVVVVVVVVVVVVVVV

conflict of law (which state’s law applies)

If suit was brought in a district with proper venue and the case is transferred to another district, the law of the transferor (first) forum controls.
* apply the choice-of-law rules of the state in which the action began; take your law with you

generally, in cases involving real property, law applied by forum court should be determined by conflict-of-law rule that would be applied by state where property is located
* except (for these, look for law of state that has most significant relationship to transaction and parties instead of determining it by where property is located):
* where document specifies application of specific jurisdiction’s law
* issues regarding effect of marriage on interest in land
* collateral issues (whether transferor perpetrated fraud)
* issues regarding mortgage note (validity of note)
* foreclosure-related issues that do not affect interest in land (mortgagee’s right to sue mortgagor for underlying debt before foreclosure)

If suit was brought in a district without proper venue and transferred to another district, the law of the transferee (second) court controls.
* DO NOT get to take your law with you; no advantage to filing in the wrong court.

How well did you know this?
1
Not at all
2
3
4
5
Perfectly
17
Q

Special rules for where venue is proper:

A

Removal

Federal district where state action was pending
VVVVVVVVVVVV

Foreign resident

Any judicial district

BUT Diverse citizenship does not exist when a suit involves stateless persons:
(1) noncitizens present in the U.S. but not citizens of a foreign country or
(2) U.S. citizens domiciled in a foreign country.

VVVVVVVVVVVV
Federal official sued in official capacity

Where defendant resides

Where substantial events occurred or property is located

Where plaintiff resides if no real property is involved in suit

VVVVVVVVVVVV
Foreign government

Where substantial events occurred or property is located

In federal district court of Washington, D.C.

VVVVVVVVVVVV
Multiparty, multiforum litigation

Where defendant resides

Where substantial part of accident occurred

VVVVVVVVVVVV
Federal Tort Claims Act

Where plaintiff resides

Where act/omission occurred

How well did you know this?
1
Not at all
2
3
4
5
Perfectly
18
Q

erie: choice of law

A

always applies in diversity case. judge will have 2 scenarios: state vs state or state vs federal

In a diversity case, a federal court applies state substantive law and federal procedure.

In federal question case, apply federal law.

VVVVVVVVVVVVVVVVVV

state substantive law

must apply the substantive law of the state in which it is located.

The substantive rules that govern conduct, (what must be proved to win a case and what defenses may be asserted);

State statutes of limitations on state causes of action;

the existence of an affirmative defense ( (NOT THE BURDEN OF PROVING THE AFFIRMATIVE DEFENSES, THIS IS PROCEDURAL AND FOR FEDERAL COURTS)

The burdens of proof on state claims or defenses; and

State rules on choice of law

VVVVVVVVVVVVVVVVVVVVV

federal procedures

Hanna v. Plumer: Anything covered by the Federal Rules of Civil Procedure is “procedure” for purposes of Erie—federal law applies, even if the substance of the claim is governed by state law.
* whether action can proceed as class action is governed by FRCP

VVVVVVVVVVVVVVVVVVVVV

no federal statute or rule on point

(commonly tested is for statute of limitations issues, this is usually substantive and state will decide)

The court must determine whether to follow state law (i.e., the matter is deemed “substantive”) or to follow federal law (i.e., the matter is deemed “procedural”).

where it is unclear whether an issue is substantive or procedural (like the deadline to commence an action under a statute of limitations= state substantive, or whether class action can commence= federal procedural) A court must then determine if a federal law directly addresses the issue.

If a conflict exists, then the federal-rule analysis provides that a valid federal rule that directly addresses the issue applies if the rule:
* is arguably procedural and
* does not abridge, modify, or enlarge a substantive right.

The twin aims of Erie:
1. To avoid forum shopping (Situations in which parties have incentives to choose/ avoid federal court based on perceived advantages and disadvantages of federal law versus state law) and
2. To avoid the inequitable administration of justice (Situations where there is a different result in state and federal courts, so winning and losing turns on the accident of diversity of citizenship)

Two common applications of Erie:
1. If the choice of the procedure would be outcome determinative (change the result), the federal court should usually apply state law to prevent forum shopping.
2. The role of the jury in federal court is entirely controlled by federal law.

VVVVVVVVVVVVVVVVVVVV

federal common law

Judge-made federal law (“federal courts have recognized”, may not be called a RULE) that overrides (preempts) any inconsistent state or local law or rule

Federal courts make federal common law ONLY when they encounter important federal interests that are not covered by statute

Examples where federal common law applies (apply federal substantive/procedural rules):
* Boundary disputes between states
* claim preclusion
* privilege (attorney-client, psychiatrist-patient)

Federal courts sometimes “borrow” state law but do so as a matter of federal common law.

VVVVVVVVVVVVVVVVVV

Follow precedent from the highest state court

If there is no precedent, predict how the highest state court would rule.

Give respectful attention to the decisions of lower state courts (not
bound).

How well did you know this?
1
Not at all
2
3
4
5
Perfectly
19
Q

fpleadings: computing time

A

Rule 6: whenever a time period is stated in days, the period excludes the day of the event that triggers the period, but it includes every day following, including intervening Saturdays, Sundays, and legal holidays.

If the last day of the period is a Saturday, Sunday, or legal holiday, then the period is extended to include the next non-weekend or non-holiday day.

How well did you know this?
1
Not at all
2
3
4
5
Perfectly
20
Q

pleadings

A

commencement of proceedings

Federal civil action is begun by filing complaint a with the court clerk.

For diversity actions, state law controls when an action is begun.

VVVVVVVVVVVVVVVVVVV

types of pleadings

(1) the complaint
* Used to state a claim for relief
* the plaintiff/defendant may file a claim against a co-party, called a cross-claim complaint
* defendant files a 3rd party complaint to implead a 3rd party defendant

(2) the answer
* filed by oppossing party in response to the complaint
* it may contain responses to allegations of complaints, affirmative defenses, and/or counter-claims

(3) the reply
* used by plaintiff to answer a complaint
* generally, party must serve a reply to an answer within 21 days after being served with an order to reply, unless the order specifies a different time

VVVVVVVVVVVVVVVVVV

requirements of a complaint

A complaint (or any pleading in which a claim is made) must include:
* a short and plain statement of the court’s subject matter jurisdiction
* a short and plain statement showing claimant is entitled to relief and
* a claim for the relief sought by pleader

(1) claim for relief
* recovery isnt limited for relief as stated in complaint, except for default judgments
* you get what you prove, no what you ask for

(2) notice pleading
* federal rules reformed pleadings: standardized pleadings across civil actions, simplified pleadings, shift emphasis from pleading to merits of case
* federal rules generally only require notice pleading: A pleading need not detail the facts of the plaintiff’s case or spell out the legal theory, Must only give fair notice of pleader’s claim (short and plain statement)

VVVVVVVVVVVVVVVVVVVV

special pleading

no particular form of words

alleged with particularity/specificity/detail

types of claims requiring special pleading:
* fraud or mistake must be specially pleaded (must show circumstances giving rise to fraud or mistake WITH PARTICULARITY)
* Claims for special damages must be specially pleaded (damages that dont ordinarily follow from wrong)

VVVVVVVVVVVVVVVVVV

recent decisions

Supreme Court cut-back on notice pleading, due to concerns about excessive discovery and fishing expeditions.

Require that the allegations in the complaint state a plausible case for recovery

Conclusory Claims: Disregard allegations not entitled to presumption of truth

Two-step inquiry:
1. Identify allegations that are “conclusory” or “mere legal conclusions”
2. Look at remaining, factual, allegations, and ask whether they add up to a “plausible” (between probable and just possible) case for recovery.

  • empowering district judges to dismiss, before discovery, complaints that they think are obviously unfounded (lack of factual detail, it tells rather than shows)
How well did you know this?
1
Not at all
2
3
4
5
Perfectly
21
Q

defense motions

A

(1) motion to dismiss

May be used to raise:
* lack of subject matter jurisdiction
* lack of personal jurisdiction (including lack of power and defects in service of process)
* improper venue
* failure to state a claim upon which relief can be granted
* failure to join a necessary party
* forum non conveniens: allows a court to dismiss an action—even if personal jurisdiction and venue are otherwise proper—if the court finds that a state or foreign judicial system is better suited to hear the dispute (the forum would be too inconvenient for parties and witnesses, and that another more convenient venue is available) The appropriate way to enforce a forum-selection clause pointing to a state or foreign forum is to seek dismissal through the doctrine of forum non conveniens.

claim of no personal jurisdiction/claim of improper venue MUST be made at the earliest opportunity (pre-answer motion if filed, if not then in the answer)

lack of SMJ may not be waived; can be raised by ANY party at ANY time, including for the first time on appeal or by the court itself

most claims (including lack of PJ and venue) are waived if NOT raised by defendant before substantial participation on the merits

issuficient service of process waived if not raised in pre-answer motion, and if no pre-answer motion is filed, in the answer.

(2) motion for judgment on the pleadings
* After the pleadings are closed, a party may move for judgment on the pleadings pursuant to Rule 12(c).
* applies when the pleadings agree entirely on the facts and only the law is in dispute
* The complaint must state enough facts to raise a reasonable expectation that discovery will reveal evidence of the necessary element.
* if facts in affidavits/discovery docs must be considered, MUST file for summary judgment instead of judgment on the pleadings

(3) motion for a more definite statement
* asks that pleading be made more specific
* defendant’s answer must be served within 10 days of service of the more definite statement.
* judges usually disfavor this motion

(4) motion to strike
* can be used to delete from pleading scandalous or prejudicial matters that are NOT relevant to the case at hand
* also used by plaintiff to strike a legally invalid defense

VVVVVVVVVVVVVVVVVV

answer

(1) in general
* used to respond to any form of complaint
* may contain responses, affirmative defenses, and counterclaims

(2) responses
* The answer must admit, deny, or state that there in sufficient knowlegde or information to form belieg as to truth or falsity of the allegations of the plaintiff’s complaint (treated like denial).
* An allegation, other than one relating to the amount of damages, will be deemed admitted if a responsive pleading is required and the allegation is not denied.
* the failure to respond constitutes an admission
* unless cured by amendment, the admission is binding in the action
* usual practice: a boilerplate denial of everything not specifically admitted

(3) affirmative defenses
* common affirmative defenses: assumption of risk, ocntributory negligence,f raud, release, statute of frauds, statute of limitations

(4) timing
* must ordinarily be served within 21 days of service to the pleading to which it responds
* if defendant waives service of process, answer is due within 60 days of when the request for waiver was sent out (90 days if the request for waiver was sent to the defendant outside of the U.S.).
* If defendant files a motion to dismiss and the court denies it, the answer is due 14 days after notice of the court’s action.

VVVVVVVVVVVVVVVVVVV

reply

Plaintiff’s answer to a counterclaim

Rules governing counterclaim apply to a reply.

How well did you know this?
1
Not at all
2
3
4
5
Perfectly
22
Q

forum selection clause

A

When transfer is sought on the basis of a forum selection clause in a contract, the clause is accorded respect
* NOT SUPPORTED FOR A MOTION TO DISMISS!
* the clause itself does not render other forums legally improper, a forum selection is set aside only upon a strong showing that transfer to the selected court would be unreasonable and unjust or that the clause was invalid for reasons such as fraud or overreaching

When venue is transferred between federal courts based on a valid forum selection clause, the transferee court must apply the law, including the choice-of-law rules, of the state in which it is located.

If the clause specifies a federal forum, it may be enforced by a motion to transfer under §1404(a).

If the clause specifies arbitration or a state or foreign forum, the common-law doctrine of forum non conveniens may instead require the action to be dismissed.

How well did you know this?
1
Not at all
2
3
4
5
Perfectly
23
Q

forum selection clause

A

When transfer is sought on the basis of a forum selection clause in a contract, the clause is accorded respect
* NOT SUPPORTED FOR A MOTION TO DISMISS!
* the clause itself does not render other forums legally improper, a forum selection is set aside only upon a strong showing that transfer to the selected court would be unreasonable and unjust or that the clause was invalid for reasons such as fraud or overreaching

When venue is transferred between federal courts based on a valid forum selection clause, the transferee court must apply the law, including the choice-of-law rules, of the state in which it is located.

How well did you know this?
1
Not at all
2
3
4
5
Perfectly
24
Q

forum selection clause

A

When transfer is sought on the basis of a forum selection clause in a contract, the clause is accorded respect
* NOT SUPPORTED FOR A MOTION TO DISMISS!
* the clause itself does not render other forums legally improper, a forum selection is set aside only upon a strong showing that transfer to the selected court would be unreasonable and unjust or that the clause was invalid for reasons such as fraud or overreaching

When venue is transferred between federal courts based on a valid forum selection clause, the transferee court must apply the law, including the choice-of-law rules, of the state in which it is located.

How well did you know this?
1
Not at all
2
3
4
5
Perfectly
25
Q

forum selection clause

A

When transfer is sought on the basis of a forum selection clause in a contract, the clause is accorded respect
* NOT SUPPORTED FOR A MOTION TO DISMISS!
* the clause itself does not render other forums legally improper, a forum selection is set aside only upon a strong showing that transfer to the selected court would be unreasonable and unjust or that the clause was invalid for reasons such as fraud or overreaching

When venue is transferred between federal courts based on a valid forum selection clause, the transferee court must apply the law, including the choice-of-law rules, of the state in which it is located.

How well did you know this?
1
Not at all
2
3
4
5
Perfectly
26
Q

denials

A
  1. the defendant may deny all allegations in plaintiff’s complaint, called general denial
  2. defendant may deny allegations of specific paragraph or averment of complaint, called specific denial
  3. defendant may deny particular portion of particular allegation, called qualified denial
  4. the defendant may deny knowledge/information if he doesnt have the knowledge/info sufficient to form belief as to truth of plaintiff’s complaint. this has full denial effect and subject to good faith requirement
  5. (judge-made extrapolation) a denial based on information and belief, allows defendant without first hand knoweldge but with enough info to believe in good faith that complaint is false, to deny it on that ground. this kind of denial is often used by large corporate defendants on whom burden of obtaining info may be great.
How well did you know this?
1
Not at all
2
3
4
5
Perfectly
27
Q

pre trial procedure: amending pleading

A

(1) types of amendments

if you want to amend complaint or other pleading
1. amending as a matter of right
2. amending by leave of court

amending as matter of right:
* can do it 1x without court permission, within 21 days of service of original pleading OR
* 21 days from being served with responsive pleading (answer) or motion asserting :
* lack of SMJ
* lack of PJ
* improper Venue
* insufficient process
* failure to state a claim
* failure to join required party
* motion for judgment on the pleadings
* motion for more definite statement
* motion to strike insufficient defense/immaterial matter from pleading

amending by leave of court:
* court’s permission. already amended or after 21 days court will let you amend if “justice so requires”

VVVVVVVVVVVVVVVVVV

A response to the amended pleading (e.g., answer) generally must be made by the later of the following deadlines:

The time remaining to respond to the original pleading—e.g., an answer is generally due within 21 days after service of process

14 days after the service of the amended pleading

VVVVVVVVVVVVVVVVVV

(2) doctrine of relation back

when amended complaint will relate back to date in time for purposes of statute of limitations to date of original pleading:
* when amended pleading arose out of the same conduct/ transaction/ occurence of original pleading
* the new party received notice of the suit within 90 days after original complaint was filed and
* the new party knew or should have known that the suit would have been brought against it but for a mistake concerning the proper party’s identity (meaning the party who should have added them did not add them because they were mistaken)

VVVVVVVVVVVVVVVVVV

(3) Amendment to Add or Change a Party

An amendment to add or change a party against whom a claim is asserted must:
* Concern the same conduct, transaction, or occurrence as the original pleading; and
* The party being added (new party) must have known or had reason to know that the action should have been brought against that party but for the honest mistake of the party making the amendment.
* Incorrect party-in-interest (plaintiff):

Example: suit brought by a parent corporation when it should have been brought by a subsidiary.

Suit should not be dismissed for having the wrong plaintiff until a reasonable time has been allowed for substitution of the real party in interest.

Amendment to substitute the real party in interest only works if the amendment concerns the same conduct, transaction, or occurrence and the original complaint gave notice to the defendant.

VVVVVVVVVVVVVVVVVV

Certification of pleadings

Verification: Most pleadings are NOT verified (not sworn to), nor do they have to be.

Certification: Pleadings and other documents—including motions and discovery requests—must be signed by attorney of record and provide the attorney’s contact information.

the signature certifies that:
* document is presented for proper purpose—not to harass, cause unnecessary delay, or needlessly increase cost of litigation
* claims, defenses & legal contentions are warranted by existing law or by nonfrivolous argument for revising or establishing law
* factual assertions have or will have evidentiary support and
* factual denials are warranted by evidence or reasonably based on belief/lack of information

  • Denials must have such support or must be reasonably based on lack of information or belief.
  • violation of certification: court can impose monetary sanctions for violations of Federal Rule of Civil Procedure 11(b). Sanction proceedings can be initiated (1) by a party’s motion or (2) on the court’s own initiative—so long as the judge issues an order to show cause.

Sanction proceedings can be initiated by the court sua sponte or by a party’s motion. When sanction proceedings are initiated by a party’s motion, the safe-harbor rule requires the party serving the motion to refrain from filing it for 21 days after serving it to allow any violation to be corrected (e.g., by dismissing a claim).

Move to dismiss or seek sanctions (Rule 11)

The attorney can be ordered to bear the costs of baseless/improper filings.

28
Q

multi-party litigation: capacity to sue, permissive joinder, compulsory joinder, intervention, interpleader

A

capacity to sue and be sued

minors (under 18) and people deemed incompetent may sue or be sued only through a guardian

Partnerships can sue or be sued as an entity if jurisdiction is based on federal question.
* if jurisdiction is based on federal question, then every partner must be listed as a party to the litigation (complete diversity applies).

VVVVVVVVVVVVVVVVVV

permissive joinder of parties

Joinder by plaintiff is governed by Rule 20.

Any number of plaintiffs may join if:
1. They assert claims arising out of the same transaction or occurrence or series of transactions or occurrences; AND
2. There is a common question of law or fact.

Any number of defendants may be joined in the same action if:
1. The claims against them arise out of the same transaction or occurrence (or series); AND
2. There is a common question of law or fact.

diversity cases:
* NO party can be joined whose presence would destroy complete diversity, even if joinder rules are otherwise satisfied

VVVVVVVVVVVVVVV

compulsory joinder of parties

Joinder is about what defendants can compel plaintiffs to do and is governed by Rule 19.

A necessary party is someone whose participation in the lawsuit is necessary for a just adjudication because:
* Absent that party, complete relief cannot be accorded to the existing parties
* The necessary party has an interest in the subject of the litigation which will be impeded by this litigation; or
* There is a substantial risk of double/inconsistent liability imposed on others if the party is not brought into the case.
* Worried about prejudice to the existing parties
* NOT A JOINT TORTFEASOR, THEY ARE NOT SUBJECT TO COMPULSORY JOINDER

Necessary parties MUST be joined if feasible.
* feasible if (1) it will not deprive the court of SMJ and (2) the court can assert personal jurisdiction over the necessary party
* bulge provision: In addition to other grounds for serving an out-of-stateparty,a necessary party may be served anywhere within 100 miles of the federal courthouse.

If a necessary party CANNOT be joined, the court decides whether to
* Continue without the necessary party (typically the case) or
* Dismiss the whole suit (rare) (i.e., the party is described as being indispensible
* The judgment is fact specific.

VVVVVVVVVVVVVVVVVVV

A motion to sever claims against a plaintigg and one of the defendants should be granted if the defendants were improperly joined.

VVVVVVVVVVVVVVVVVVV

intervention

Outsider who volunteers to enter a lawsuit

Chiefly a tool for people hoping to join as Plaintiffs; governed by Rule 24.

Types of intervention:
* Intervention as of right—Is available when the outsider claims an interest in the subject matter of the lawsuit that, as a practical matter, may be compromised by the disposition of the pending action.
* Permissive intervention—May be allowed when ever there is a common question of law OR fact between the intervenor’s claim and the main claim (very relaxed standard, must ask the court’s permission- matter of court’s sound discretion).

Both types must be timely: “reasonable promptness”

There is no supplemental jurisdiction for either kind of intervention when
jurisdiction is based on diversity.
* Diversity case: intervenors must satisfy complete diversity AND have a claim that exceeds $75,000.

VVVVVVVVVVVVVVV

interpleader

Used to resolve the problem of competing claims to the same property (could be like insurance proceeds)

Designed to avoid inconsistent obligations or multiple claims; possessor of the stake can avoid multiple liability by joining the claimants to litigate the ownership of the stake among themselves

Property at issue is called the stake
* May be real or personal, tangible or intangible

Person holding the property is the stakeholder

Person claiming the property are claimants

If stakeholder claims a right to keep the property, then the person is called a stakeholder claimant

Stakeholder can invoke interpleader either as a plaintiff OR a defendant:
* Stakeholder-plaintiff sues all the claimants as defendants.
* Stakeholder-defendant is one who has been sued by a claimant; all the other claimants are joined as plaintiffs.
* Under the federal interpleader rule, unlike the federal interpleader statute, diversity of citizenship is tested between the stakeholder and the other claimants. There is no requirement under the rule, unlike the statute, that there be diversity between at least two of the claimants.

rule interpleader

Authorized by Rule 22

Remedy available in a lawsuit otherwise within the court’s personal jurisdiction

requires that
1. the action involve a federal question OR
2. the amount in controversy exceed 75k and complete diversity exists between stakeholder and claimants- they must be citizens of different states

statutory interpleader

Comes from Federal Interpleader Act and only applies in federal court

Has special provisions:
* Special jurisdictional amount: Need only be $ 500
* SMJ based on minimal diversity (at least two claimants of different states)
* Authorizes nation-wide service of process
* Venue is proper in any district where any claimant resides.
* deposit – requires the stakeholder to deposit the property at issue with the court or post a bond in an amount determined by the court

29
Q

multi-claim litigation: joinder

A

As between the same plaintiff and the same defendant, all claims may be joined (need not be related)

Diversity case: Plaintiff can aggregate all claims against the same defendant to exceed the jurisdictional minimum

Federal question case: If diversity is lacking for additional state-law claims, additional state-law claims can be joined only if they are covered by supplemental jurisdiction (i.e., arise from the same transaction or occurrence as the original claim).

30
Q

multi-claim litigation: counterclaims

A

Claim by an opposing party, usually by the defendant against the plaintiff

Types of counterclaims: permissive and compulsory

compulsory counterclaims

A counterclaim is compulsory if it arises out of the same transaction or occurrence as the claim to which it responds.

A compulsory counterclaim is lost (waived) if not pleaded in the current action.

But when an action is dismissed before a defendant’s answer is filed (could have been dismissed based on motion from the defendant), any counterclaims that would have been compulsory in an answer are not waived.

supplemental jurisdiction will generally be available for compulsory counterclaims.
* If there is SMJ over the main claim, there is SMJ over a compulsory counterclaim.
* Diversity case: If counterclaim raised by a defendant, then it is not barred by any of the plaintiff’s diversity supplemental jurisdiction exclusions; Amount of compulsory counterclaim USUALLY does not matter
* Statute of Limitations (SOL): Filing of the original complaint tolls the SOL for the original claim AND any compulsory counterclaim(s).

permissive counterclaims

Counterclaim that does not rise out of the same transaction or occurrence

May be pleaded now OR raised later

Requires an independent jurisdictional basis: Must be a federal claim OR there must be complete diversity of citizenship with $75,000+ in issue.

Statute of Limitations: A permissive counterclaim must be timely as of the date it is filed or it’s time-barred.

31
Q

multi-claim litigation: cross-claims

A

Claims asserted against a co-party
* Plaintiff v. co-plaintiff, defendant v.co-defendant

Must arise out of the same transaction or occurrence as the original claim or a counterclaim

Cross-claims are never compulsory.

32
Q

multi-claim litigation: Impleader (Third-Party Practice)

A

Impleader is a device by which the defendant brings into the suit someone who is or maybe liable to the defendant for all or part of the plaintiff’s claim against him

application

Impleaded party: Third party defendant

Original defendant: Third party plaintiff as against the third-party defendant

Often applies to indemnification contracts

Also often applies to contributuon among joint tortfeasors

Reasoning: Impleader is not allowed because the other tortfeasor may be liable to the plaintiff; Impleader is allowed because, the second tortfeasor may be liable to the defendant for part of the defendant’s liability to the plaintiff.

subject matter jurisdiction

Diversity cases: Impleader comes within the court’s supplemental
jurisdiction.
* Amount of claim against third-party defendant does NOT matter
* Only extends to claims by the defendant against a third-party defendant

Supplemental jurisdiction does nog extend to claims by the original plaintiff against the impleaded third-party defendant (because those would be claims by a plaintiff against a person made party under Rule 14).

personal jurisdiction

Bulge provision: Under “100-mile bulge rule” a federal court acquires personal jurisdiction over parties (1) added to the suit through impleader or required joinder and (2) served with process within 100 miles of court.

33
Q

multi-claim litigation: class actions

A

Class action certification is proper if :
1. Numerosity (i.e., too many parties to be joined conventionally, usually over 40);
2. common questions of law or fact;
3. typicality of claims by the class representatives; and
4. adequacy of representation

and

The case is one of the following types of class actions:

  1. Prejudicial risk – when separate actions would create a risk of inconsistent decisions regarding the parties or impairing absent class members’ interests (notice is not required because class members do not have a right to opt out.)
  2. Common question – when common questions of law or fact predominate over individual questions and a class action is the best method to fairly and efficiently adjudicate the dispute (adequate notice must be provided to all class members because they have the right to opt out of the class action and sue on their own behalf)
  3. Final equitable relief – when injunctive or declaratory relief is appropriate because the opposing party’s actions generally apply to the whole class (notice is not required because class members do not have a right to opt out.)

VVVVVVVVVVVVVVVVVVVVVV

Burden is on proponent of class certification to establish that these prerequisites aremet.

An appellate court may hear an appeal from a district court order that grants or denies class action certification. If the appellate court permits the appeal, the district court proceedings are stayed pending the appeal only when ordered by the district court or the appellate court

*dismissal or compromise (Settlement) *

requires judicial approval

diversity jurisdiction
Named representatives must be completely diverse from the defendants AND at least one plaintiff must have a claim worth over $75,000; ourt can acquire subject-matter jurisdiction over class members’ claims that do not exceed $75,000 through supplemental jurisdiction

class action fairness act of 2005
Allows very large class actions, involving at least 100 members with more than $5 million at stake

VVVVVVVVVVVVVVV

For a class action that is certified under Rule 23(b)(1) or (2), notice to the class members, other than the named representatives, is at the discretion of the court. The court may order that appropriate notice be given, which often takes the form of publication notice.

34
Q

scope of discovery

A

mandatory disclosures

in general

Designed to streamline discovery

Requires disclosure without having to wait for a discovery request

three stages

initial disclosures
* names and addresses (contact info) of people with potentially discoverable info (THIS DOES NOT REQUIRE WITNESSES WHO ARE ONLY BEING USED FOR IMPEACHMENT PURPOSESTO BE DISCLOSED! JUST SUBSTANTIVE IS REQUIRED!
* Copies or descriptions of relevant documents or things
* Computation of the damages claimed
* Applicable insurance agreement(s)

disclosure of expert witnesses
* at least 90 days before the date set for trial.
* However, if the failure to disclose in a timely manner is harmless, the party who failed to timely disclose can still use testimony at trial
* Names of expert witnesses who will be called at trial
* Qualifications, publications, opinions, information on which they will base their
opinions, other cases in which they have testified, and compensation

pretrial disclosures
* 30 days before trial
* The name, address, and phone number of each witness, separately identifying witnesses expected to testify at trial and those who may be called if the need arises
* The designation of witnesses whose testimony will be presented by deposition, including a transcript of the pertinent parts of any deposition that was not taken stenographically (i.e., by shorthand)
* All documents and exhibits, separately identifying items expected to be offered as evidence and those that may be offered if the need arises

  • Any objections must be made within 14 after disclosure or they are waived unless excused by the court for good cause.

VVVVVVVVVVVVVVVVVVVV

Scope of discovery

in general

Applies to all six discovery devices

general rule is relevance
* the scope of discovery is not limited to admissible evidence.

proportionality

Limited to matters that are proportional to the needs of the case

Courts consider the importance of the issues, amount at stake, parties’ resources and access
to information, value of discovery in the case, and, most importantly, whether the burden or expense of discovery outweighs its likely benefit .

excpetions to discovery

evidentiary privilege
* Anything covered by an evidentiary privilege is not discoverable.

work product rule
* Basic idea: Preparation for litigation by one party cannot be discovered by the other.
* The attorney work product rule protects:
1. document and things (not information)
2. Prepared in anticipation of litigation or for trial; (Documents prepared before the cause of action arose (e.g., ordinary business records) are NOT protected by the work product rule.)
3. By or for another party or the party’s representative.

Result: Creates a qualified immunity from discovery that can be overcome only if the party seeking discovery shows:

special cases
Notwithstanding the above, you can:
* always get a copy of your own statementswhether you are a party or a mere witness.
* never discover the mental impressions of the lawyer.

experts

Distinguish between experts who will be called to testify and those who will not

Other side can a;always discover the report of the testifying expert.

if expert is going to testify, the other side, infairness, has to prepare for cross-examination.
* Can discover the amount of compensation, and the facts, data, and assumptions provided to the expert by the lawyer

If expert is NOT going to testify, no discovery absent (?) circumstances

protective orders

For good cause shown, court can basically do anything, in its discretion, that justice may require to protect a party from annoyance, embarrassment, or undue expense during discovery

35
Q

discovery devices: six discovery devices

A

(1) oral deposition

  • Questions are asked and answered orally and under oath.
  • Limited to 10 depositions, unless the court allows more
  • Each is limited to one day of 7 hours, unless the court allows more.
  • -To compel the attendance and testimony of a nonparty deponent, the deposing party must serve the nonparty deponent with a subpoena
    The subpoena may be served and the deposition conducted without the court’s leave (permission) or the parties’ stipulation unless:
  • the deposition exceeds the 10-deposition limit (which includes both oral and written depositions)
  • the deposition is sought before the parties’ initial planning conference or
  • the deponent was already deposed in the action.
  • Leave of court is always required when deponent is in prison.
  • Any kind of notice suffices for the deposition of a party; but a deposition of a nonparty (“mere”) witness requires a subpoena.
  • Subpoena duces tecum: requires the deponent to bring specified documents or things
  • To depose an organization, serve notice or subpoena on organization; organization then
    selects person who will be deposed
  • Can be taken any time after the party has made mandatory disclosures
  • May be taken before any notary public who is not otherwise disqualified (usually
    stenographer)

(2) written deposition
* Questions asked in writing are delivered to a hearing officer who asks the questions orally and the witness answers orally under oath.
* Rarely used because they are so inflexible

(3) interrogatories
* Questions asked in writing to be answered under oath in writing
* May only be used against a party
* Discovery requests (like interrogatories) generally cannot be served until the parties have held an initial planning conference to arrange for initial disclosures and prepare a discovery plan.
* Presumptively limited to 25 interrogatories, unless the court allows more
* Responses required within 30 days
* May respond by producing business records if the answer can be ascertained and the burden of deriving the answers is substantially the same for the party serving the interrogatories as for the responding party

(4) Discovery and Inspection of Documents and Land
* Called a request to produce and permit inspection
* Applies only to documents, things, and land under the control of a party
* The thing to be produced and inspected must be described with particularity
* Response is due within 30 days

(5) Physical and Mental Examination - Requirements:
* Available only against a party
* Only permitted when the party’s physical or mental condition is in controversy; and
* Only for good cause shown

(6) request for admission
* Used to streamline the litigation
* Failure to respond within 30 days is an admission
* Admissions are binding in litigation, but have no preclusive effect (only binding in the current lawsuit; cannot be used against the party in any future proceeding).

36
Q

quashing or modifying subpoena

A

required

court must quash or modify subpoena that
* fails to allow reasonable time to comply
* requires person to comply beyond geographical limits
* requires disclosure of PRIVILEGED OR PROTECTED matter (
* subjects person to undue burden

permitted

court may quash or modify subpoena that requires disclosing:
* unretained expert’s opinion or information that (1) does not describe occurences in dispute and (2) results from expert’s study not requested by party
* trade secret or confidential research, development, or commercial information

37
Q

discovery devices: use of depositions at trial

A

Discoverability does not equal admissibility.

Deposition of an adverse party is admissible as an admission of a party opponent.

Deposition of a mere witness can be used to impeach that witness.

Deposition of a witness who does NOT testify can be used if the witness is dead, beyond the
court’s subpoena power, or otherwise unavailable.
* Can also be used if the witness is more than 100 miles from the place of trial.

38
Q

discovery devices: enforcement sanctions

A

The court can immediately impose sanctions in three instances of complete default:
1. Failure to attend one’s own deposition;
2. Failure to respond to interrogatories; and
3. Failure to respond to a request for documents or things.

There is no way that you can, in good faith, fail to show up or respond.

In other cases, the party seeking discovery must go to court and obtain an order compelling discovery.
* the court should first issue an order to compel based on the party’s request before imposing sanctions.
* If that order is disobeyed, the court may do what it deems is necessary.

When ruling on a motion to compel, the court may award reasonable expenses associated with the motion based on the following scenarios:
* If the motion is granted, the court must award the movant the reasonable expenses incurred in making the motion unless an exception applies.
* If the motion is granted in part and denied in part, the court may apportion the reasonable expenses as it sees fit.
* If the motion is denied, the court must award the nonmovant the reasonable expenses incurred in opposing the motion unless the motion was substantially justified or the award would be unjust.

39
Q

discovery devices: electronically stored information

A

Normally should be preserved

If lost through unreasonable conduct of a party, the court may order measures to cure the prejudice to the other party.

If the material was destroyed with intent to prejudice the other party, the court may instruct the jury to presume the information was unfavorable, or even order the end of the litigation.

40
Q

pre trial conference

A

Must be attended by the attorneys who will conduct the trial

Must file a pretrial statement detailing claims and defenses, itemization of damages, requests for stipulations and admissions, list of all witnesses and exhibits, etc.

Failure to comply usually means that the attorney pays the costs and the other side’s attorney’s fees.

During these conferences, the judge has broad discretion to consider and act on a wide range of issues, including:
* establishing a time limit for each party’s presentation of evidence
* limiting each party’s use of expert testimony and
* ruling on the admissibility of evidence

41
Q

pretrial amendments

A

under FRCP, court should modify a pretrial order (like adding a witness or amending order to add another claim) only to prevent manifest injustice.

42
Q

motions to terminate: termination without trial

A

(1) devices
* judgment on the pleadings
* default judgment (defendant has not shown up);
* voluntary dismissal dismissal (dismissal without prejudice);
* involuntary (dismissal with prejudice);and
* summary judgment

VVVVVVVVVVVVVVVVVVVV

(2) voluntary dismissal
* Ordinarily without prejudice (Party whose claim is dismissed can bring that claim again in a new
lawsuit.)
* Absent the parties’ stipulation, a party may voluntarily dismiss a crossclaim, counterclaim, or third-party claim without a court order by unilaterally filing a notice of dismissal (1) before a responsive pleading is served (answer or motion for summary judgment) or (2) if a responsive pleading is not served, before evidence is introduced at a hearing or trial.
* The defendant’s motion to dismiss (for lack of jurisdiction/improper venue/failure to state a claim upon which relief can be granted.) before filing an answer is NOT a responsive pleading does NOT cut off the right to a voluntary dismissal.
* After a defendant has filed an answer or motion for summary judgment, or if the plaintiff has already voluntarily dismissed once, plaintiff must seek leave of court for dismissal without prejudice.
* Plaintiff will have to pay the costs if she ends up later filing the same action against the same defendant.

VVVVVVVVVVVVVVVVVVVV

(3) involuntary dismissal
* Typically with prejudice
* Involuntary dismissal for lack of jurisdiction, improper venue, or failure to join an
indispensible party is without prejudice.
* In all other cases, involuntary dismissal is with prejudice
* Dismissal with prejudice is an adjudication on the merits, which means that, under federal law, it is given full res judica (preclusive) effect, which bars any attempt at re-litigation of the same claims.
* May be imposed for plaintiff’s failure to prosecute or for failure to comply with the FRCP or any court order
* Standard for appellate review is abuse of discretion.

VVVVVVVVVVVVVVVVVVVV

(4) summary judgment
* This motion may be filed at any time until 30 days after the close of discovery
* Compare with motion to dismiss for failure to state a claim upon which relief can b egranted (12(b)(6)):
1. Failure to state a claim under 12(b)(6) tests only the legal sufficiency of the plaintiff’s claim
2. Summary judgment can be used to test both the facts AND the law.

  • Partial summary judgment: summary judgment can be granted for the entire case, or only for certain parties, certain claims or defenses, or certain issues
  • Standard: There is no genuine dispute as to any material fact and the moving party is entitled to judgment as a matter of law.
  • “No genuine dispute” means that no reasonable jury could find for the nonmoving party.
  • Must be supported or opposed by particular materials in the record, such as:
    1. depositions
    2. answers to interrogatories
    3. Affidavits
    4. Stipulations
  • Materials in the record must generally be sworn statements to be considered in deciding a motion for summary judgment.
  • Pleadings are generally NOT sworn statements, but if verified, or taken under oath, they become an affidavit as well as a pleading.
  • Mere assertion or denial of a fact in a pleading does NOT create a genuine dispute.
  • Sworn statement ordinarily must be based on personal knowledge

Once the movant satisfies this initial burden, the burden shifts to the nonmovant to avoid summary judgment by either:
* showing specific disputed facts through affidavits, declarations, discovery, or other materials containing admissible evidence or
* requesting that the court postpone consideration of the motion until additional discovery can take place (this works when the motion for SJ was filed before discovery), and including an affidavit or declaration that describes the desired discovery and why it is needed

43
Q

entering default judgment

A

A default refers to a defendant’s failure to timely serve an answer to a lawsuit, which is generally due within 21 days after the defendant is served with process.

When the plaintiff shows this failure to the court clerk, the clerk must enter the defendant’s default into the record of the case

A default judgment can then be entered by the clerk or the court

The clerk must enter a default judgment when:

  1. the plaintiff’s claim is for a sum certain (i.e., a specified or set amount) OR a sum that can be made certain by calculation
  2. the plaintiff’s request for default judgment includes an affidavit establishing the amount due
  3. the defendant failed to appear—i.e., did not file a motion or otherwise act before the court—and
  4. the defendant is not legally incompetent or a minor.
44
Q

right to jury

A

(1) right to jury trial

  • 7th Amendment right to trial by jury depends on the division between law (juries) and equity (no juries) as of 1791.
  • Remember: there is no right to jury trial for equity issues such as injunction, specific performance, or admiralty issues
  • where a case involves both legal and equitable claims that share common issues of fact the court must first hold a jury trial on the legal claim and then a nonjury trial on the equitable claim, relying on the jury’s findings of fact on the legal claim to do so
  • Claims for damages are the classic remedy at law and trigger the right to jury trial
  • When legal and equity issues overlap in one lawsuit, try the legal issues first, no matter the order in which they arose.

VVVVVVVVVVVVVVVVVV

(2) demand for trial by jury

  • May be made by any party either in a complaint or separate motion
  • Must be made not later than 14 days after service of the last pleading directed at the issue
  • Often made in the complaint or answer
  • if no jury demand is made, issues on which a jury trial is not properly demanded are to be tried by the court. but the court may, on a motion, order a jury trial on any issue for which a jury might have been demanded. what this means is the court may grant the request for a jury trial, even after 14 day limit passed, at the court’s discretion. (courts have broad discretion in making this decision)

VVVVVVVVVVVVVVVVVV

(3) jury selection

  • Minimum of 6 jurors (unless the parties stipulate otherwise)
  • maximum of 12 jurors
  • All must participate in verdict unless dismissed for good cause.
  • Each litigant gets 3 peremptory challenges (no need to explain or justify).
  • Cannot be used for reasons of race or gender
  • A juror must participate in the verdict unless excused by the court (THE JUDGE) for good cause during trial or after jury deliberations have begun. Good cause exists when the juror has an illness or family emergency or has committed juror misconduct that might cause a mistrial.

VVVVVVVVVVVVVVVVVV

(4) bench trial

  • Judge is required to make findings of fact on the record and state conclusions of law.

VVVVVVVVVVVVVVVVVV

(5) jury instructions

Parties may request specific instructions by close of evidence unless earlier, reasonable deadline set by court

may file request after close of evidence when:
1. request relates to issue that party could not have reasonably anticipated by deadline or
2. court permits late submission related to any issue

Judge must inform the parties of his actions on those requests and the court’s proposed instructions before instructing jury and could be BEFORE OR AFTER closing arguments

court may consider objection made after court instructs jury if instruction constituted plain error
* Court must provide an opportunity to object to proposed instructions
* court may consider objection made after court instructs jury if instruction constituted plain error
* party’s objection must be on the record (preserved) at court-appointed time or promptly after learning of proposed instruction for stated reasons (so the appellate court can follow the argument), and made BEFORE the jury retires to deliberate.
* Only a timely objection preserves the issue for appeal.
* court may instruct jury at any time before discharge

VVVVVVVVVVVVVVVVVVV

(6) jury verdict

  • Jury’s verdict must be unanimous, unless the parties have stipulated otherwise.
  • where there in inconsistency between jury’s answers to the interrogatories and verdict (like where they find party is contributorily neglgient in state where it would barr recovery, but then grants that contributorily neglgient party), the court MAY but it not REQUIRED to order a new trial. court is permitted to direct jury to deliberate further OR order a new trial.
45
Q

types of jury verdicts

A

general verdict:
* jury finds in favor of, and awards any damages to, particular party
* no statement on jury’s findings of fact

general verdict with answers
* jury finds in favor of, and awards any damages to, particular party
* jury answers questions on specified factual issues

special verdict
* jury answers questions on specified factual issues
* judge applies law to jury’s answers and enters appropriate judgment

VVVVVVVV

In a jury trial, the judge has complete discretion to decide which verdict to use—general verdict, general verdict with answers, or special verdict.

46
Q

jury view of property, matter, or thing in controversy

A

A jury view of the place in question or any property, matter, or thing relating to the controversy between the parties is not permitted unless evidence is introduced showing that the place or thing to be viewed has not changed since the events alleged in the pleadings.

47
Q

bench trial

A

In a case tried to the court (ie, a bench trial), the judge serves as both the finder of fact and interpreter of the law.

As a result, Federal Rule of Civil Procedure (FRCP) 52(a) requires that the judge provide factual findings and legal conclusions (orally or in writing) on the record after the close of evidence. This allows the parties and the appellate court to have a clear understanding of the judge’s reasoning.

If a judge fails to provide these required findings and conclusions, the appellate court should reverse the trial court’s judgment.

48
Q

motions: JMOL

A

judgment as a matter of law

Essentially a motion for summary judgment during/after the trial has begun.

Can be raised by Def after P rests or after both sides rest but before case goes to jury

Standard: Viewing the evidence in the light most favorable to the non-moving party, the evidence cannot support a verdict for that party, and the moving party is therefore entitled to judgment as a matter of law.
* Rule 50 provides the court must find that there is insufficient evidence for a jury reasonably to find for the nonmoving party.

Credibility of a witness: If the case turns on this issue, then there is a dispute as to an issue of material fact and the motion for JMOL will be denied; the issue is for the jury.

Made by defendant at the end of plaintiff’s case or by either party at the close of all the evidence

VVVVVVVVVVVVVV

renewed motion for judgment as a matter of law

A motion for JMOL made at the close of all the evidence and denied by the court may be renewed within 28 days of the jury returning a verdict
* COURT CAN NOT EXTEND DEADLINE FOR JMOL

Standard is the same: The evidence cannot support the jury’s verdict (insufficient evidence to support verfict) and the moving party is therefore entitled to judgment as a matter of law.

Prior motion is required: It is a condition precedent to a post-verdict motion that the motion for JMOL had been made at the close of all evidence.

49
Q

motions for a new trial

A

Usually made with a Renewed Motion for JMOL; both must be made no later than 28 days after the entry of judgment

Not restricted in the same way as a Renewed Motion for JMOL

A new trial may be granted in the sound discretion of the court for many reasons, including:
* Legal errors
* Newly discovered evidence
* Prejudicial misconduct by a lawyer, party, or a juror
* Judge concludes that the verdict is against the great weight of the evidence, either in substance of the verdict or amount of damages awarded

trial court can order new trial OR direct jurors to deliberate further, if after polling jurors individually, there was a lack of unanimity

Remittitur: If the court determines that a verdict was seriously excessive, then it may offer a remittitur to reduce the verdict and grant a new trial on the condition that the remittitur is not accepted.

However, it may not give the opposing party the option to avoid a new trial by agreeing to an increase in the amount of damages (i.e., additur).

The new trial order must specify the reasons for the new trial to allow for appellate review.
* If the reason is a question of law (legal conclusions, content of jury instructions), it is reviewed de novo.
* mixed questions of law and fact are reviewed de novo. generally, whether a set of facts meets a legal definition is considered to be a mixed question of law and fact.
* Other grounds for a new trial are reviewed for abuse of discretion.

COURT CAN NOT EXTEND DEADLINE FOR JMOL, AMENDED/ADDITIONAL FINDINGS, NEW TRIAL, ALTER/AMEND JUDGMENT, OR JUDGMENT FROM RELIEF

50
Q

appellate standard of review: de novo

A

no deference to trial judge’s legal determination

reverse if reasonably believe trial judge misinterpreted law

applies to: pure legal issues like legal conclusions and content of jury instructions

51
Q

altering jury’s award of damages: remittur vs additur

A

Remittur:

Defendant’s request to reduce jury award of excessive damages;

Plaintiff must be offered choice between:
* reduced damages or
* new trial on damages

VVVVVVVVVVVVVVV

Additur

Plaintiff’s request to increase jury award of inadequate damages

Never allowed in federal court

52
Q

motion to terminate without trial

A

Some preliminary motions are no longer relevant on appeal once there has been a full trial.

A motion to dismiss for failure to state a claim, motion for judgment on the pleadings, motion
for summary judgment, or motion for JMOL made before the jury retires are waived if the moving party proceeds with trial once the motions are denied.

Appellate review is based on the judgment rendered after full trial, not the earlier motion.

53
Q

appeals:

A

(1) final judgment rule

Ordinarily, appeals are available only from a final judgment
* A final judgment resolves ALL the claims of ALL the parties.

When an action involves multiple claims or parties, a district court may enter final judgment as to fewer than all claims or parties if it expressly determines that there is no just reason for delay.
* for no just reason for delay to exist, the court generally require that the adjudicated AND non-adjudicated claims be separate and distinct (or else there is technically no final judgment)
* However, if this express determination is not made, any court order that disposes of those claims or parties is not immediately appealable.

Partial final judgment: The court may enter a final judgment on some claims by expressly designating the order as final.

Judgment becomes final when entered by the court on the court’s docket (i.e., not when announced).

Notice of appeal must be filed in the trial court within 30 days of entry of judgment.

VVVVVVVVVVVVVVVVVVV

(2) interlocutory appeals

interlocutory orders immediately appealable as of right

  • Injunction (anything affecting property, too) (grant/denial)
  • Certification by district court
  • Class action certification
  • Appointment of receiver
  • Admiralty case
  • Collateral-order doctrine
  • Bankruptcy cases (certain orders)
  • Mandamus (petition for writ)
  • Patent infringement order (only accounting left)

mnemonic: In Certain Circumstances, An Appeal Can Be Made

discretionary interlocutory review

Any interlocutory order is appealable with leave of courts
* BOTH the trial and appellate courts must agree to allow the appeal.

The trial court issues a certificate for interlocutory appeal stating:
* That the issue involves a controlling of law with substantial ground for difference of opinion; and
* That the immediate appeal may materially advance termination of the litigation.

The appellate court must then agree to accept the appeal.

VVVVVVVVVVVVVVVVVVVV

(3) class actions

An appellate court has discretion to permit an appeal from an order granting or denying class action certification if the petition for such an appeal is filed with the clerk of the appellate court within 14 days after the order is entered.

VVVVVVVVVVVVVVVVVVVV

(4) collateral-order doctrine

provides a narrow exception to the final-judgment rule. Under this doctrine, an interlocutory order will be characterized as final and immediately appealable if three elements are met:
1. The order conclusively resolves an important issue (the water and sewer authority’s immunity).
2. That issue is separate from the merits of the underlying claim (whether the water and sewer authority breached the contract).
3. The order cannot be effectively reviewed on appeal from a final judgment (the water and sewer authority’s claim of immunity will be ineffective once the case has gone to trial).

Examples of collateral orders (District court orders):

  • denying state 11th Amendment immunity
  • denying government official/employee immunity
  • vacating attachment of vessel
  • refusing to require security bond pursuant to state law
  • remanding action to state court based on abstention

VVVVVVVVVVVVVVVVVVVV

(5) writ of mandamus

A petition for a writ of mandamus is an exception which allows a party to seek appellate review of a district court’s order BEFORE a final judgment has been entered.

It requests that an appellate court direct a district court to do or refrain from doing something when exceptional circumstances clearly exist and no other adequate remedy is available.

As a result, this writ is appropriate only in extremely limited situations, for example:
* when a district court clearly exercises unlawful jurisdiction
* when a district court clearly usurps another branch’s power and threatens separation of powers
* to protect a party’s Seventh Amendment right to a jury trial

VVVVVVVVVVVVVVVVVVVV

(6) standards of review

questions of law (elements of a claim)

  • Appellate review is de-novo
  • Did the trial court make an error?
  • Was the error prejudicial?

findings of fact

  • Jury verdicts must be affirmed if supported by substantial evidence.
  • Judge’s findings of fact must be affirmed unless they are clearly erroneous.

matters of discretion

  • Many issues decided by lower courts (continuances, new trials, discovery limitations, evidentiary discovery rulings)
  • Standard of appellate review is abuse of discretion
  • Any reasonable decision will be upheld.

appellate court does not have jurisdiction over ana ppeal from a party’s sanctions claim, but it does have jurisdiction over a party’s appeal from that denial and from the judgment entered in favor of the other party

example: appealling summary judgment (not an interlocutory order) where a fed claim was dismissed but not state claim, and the claims closely parallel, it will not be granted because claims are different= does not satisfy the “no just reason for delay element”

54
Q

appellate standard of review: clear error

A

Highly deferential to trial judge’s factual findings

Reverse if no reasonable judge would have made this finding

Factual issues in bench trials:
credibility of witnesses
factual determinations

55
Q

appellate standard of review: substantial evidence

A

highly deferential to jury’s factual findings

reverse if no reasonable jury would have made this findings

applies to: factual issues in jury trial, like credibility of witnesses and jury verdicts

56
Q

appellate standard of review: abuse of discretion

A

highly deferential to trial judge’s discretionary decisions

reverse only if decision was unreasonable or arbitrary

applies to: discretionary rulings by judge like grant/denial of new trial and admissibility of evidence

57
Q

full faith and credit

A

Courts in the U.S., both state and federal, must give full faith and credit to judgments rendered by courts of other states, provided that the rendering court had jurisdiction

58
Q

claim preclusion (res judicata)

A

In determining the claim-preclusive effect of a prior federal judgment in an action based on diversity jurisdiction, federal common law requires that the law of the state in which the forum court rendered its judgment be applied, unless the state law is incompatible with federal interests
* meaning, if forum court (the first court) which made judgment accepts non-mutual issue preclusion, then third party can bring claim in ANY OTHER COURT and use the law from the FORUM STATE which allowed for the use of non-mutual issue preclusion (UNLESS, STATE CONFLICTS WITH FEDERAL, YOU WILL SEE THE FACTS SAY STATE A ALLOWS, BUT STATE B DOES NOT)

vvvvvvvvvvvvvv

Preclusion consists of two distinct doctrines:
* claim preclusion (res judicata)
* issue preclusion (collateral estoppel)

Ask two questions IN THIS ORDER:

  1. Is the claim in the second suit precluded by the prior adjudication? If yes, the inquiry ends.
  2. If not, are there any issues in the second suit precluded by the prior adjudication?

VVVVVVVVVVVVVVVVVVV

claim preclusion

bars identical parties from relitigating identical claims after the entry of a valid final judgment on the merits (like summary judgment)
Claims are identical if they:
(1) arise from the same transaction, occurrence, or series thereof and
(2) could have been raised in the first action because the claim existed and could have been joined to the first action.

THEY CAN BE DIFFERENT CLAIMS, THAT DOES NOT MATTER. WHAT MATTERS ARE THE ABOVE ELEMENTS

Factors considered in determining what constitutes the same transaction or series thereof include:
* whether the facts are related in time, space, origin, or motivation
* whether the facts form a convenient trial unit and
* whether treating the facts as a unit conforms to the parties’ expectations.

VVVVVVVVVVVVVVVVVVVV

three requirements

  1. There must have been a final judgment on the merits in the first suit;
  2. The second suit must be between the same parties or their successors in interest; and
  3. The second suit must involve the same claim or cause of action.

VVVVVVVVVVVVVVVVVVVV

final judgment on the merits

Includes a default judgment, summary judgment, and dismissal with prejudice

NOT necessary that there had been a trial

VVVVVVVVVVVVVVVVVVVV

re-ligitigation between the same parties

Both parties must have been parties to the first action or successors in interest to the original parties.

A successor in interest stands in the shoes of the original party an distreated, for preclusion purposes, as the same party.

Examples of predecessors and successors in interest include:
* The assignor and assignee of a claim;
* A decedent and the executor of the estate; and
* The executor of an estate and persons who claim under the will.

Class action: Each member is generally bound by the judgment; they are considered to have had their opportunity in court even if not a named representative.
* Damages class action certified under Rule 23(b)(3), a valid judgment does not bind class members who opted out of the lawsuit.

VVVVVVVVVVVVVVVVVVV

ALL legal theories to recover for harm arising out of a single transaction or occurrence count as the same claim for purposes of claim preclusion.

Unless state law provides otherwise, if both contract and tort claims seek redress forthe same harm, they are the same claim.

Also bars claims that could have been brought in the first case but were not

Installment Sales: Creditor must sue for all that is “due and owing”at the time of the suit.
* All debt owed at the time of the action is one claim even if it was due in 3 or 4 separate payments.
* Future debts or obligations are another claim.

59
Q

issue preclusion (collateral estoppel)

A

three requirements
1. The same issue of fact must arise in two suits;
2. That issue must have been actually and necessarily decided in the first suit, by a valid and binding judgment; and
3. The party to be precluded must have been a party to the first suit.

basically: bars the relitigation of issues of fact or law that were actually litigated, determined, and essential to a valid final judgment in a prior lawsuit.
* essential issue= if it provided a clear and necessary component of the ultimate decision or, stated differently, if the final judgment depended on the determination of that issue.

VVVVVVVVVVVVVVVVVVVVVVV

(1) Same issue of fact

It does not matter if the two suits involve entirely different claims, so long as there is a common issue of fact.

VVVVVVVVVVVVVVVVVVVVVVV

(2) actually and necessarily decided

Only applies to issues actually litigated, not to those that might have been litigated

Default judgment: Results in full claim preclusive effect, but no issue preclusive effect because nothing was actually litigated

To be actually litigated, must be some sort of adversarial stance by the parties and an actual
resolution of the issue by a factfinder or judge

VVVVVVVVVVVVVVVVVVVVVVV

(3) party to be precluded must have been party to the first suit or a successor in interest

The party invoking preclusion does not need to have been a party to the prior action, nor in any way involved in the action.

Whether a succession of mutuality could invoke issue preclusion offensively is not clear.

VVVVVVVVVVVV

Issue Preclusion Types

  • mutual: issue preclusion in second action asserted by parties to the first action against other parties to the first action
  • nonmutual: issue preclusion in second action asserted by nonparties to the first action against parties to the first action

A court will look to the court that entered the first judgment to determine which issue-preclusion rule applies to the second action. A judgment entered by a federal court means federal preclusion law applies, while a judgment entered by a state court means state’s preclusion law applies
* a federal court sitting in diversity will generally paply the prec;usion law of the state in which it sits unless state law is incompatible with federal interests

example: if state A enters the first judgment, State A’s preclusion law applies in second action in federal court. if state A does not permit nonmutual issue preclusion then the nonparty to the first action cannot argue the state court judgment bound the federal court on the same issue.

60
Q

post trial: Alteration of or Relief of a Judgment–> correction of a judgment

A

Rule 60(a) allows a court to correct a clerical or other mistake resulting from oversight or omission whenever one is found in a judgment, order, or other part of the record.
* not used to correct an erroneous interpretation of law.

An example of such a mistake would be when a court meant to enter a judgment for the plaintiff for $100,000, and it appeared in the written judgment as “$10,000.”

The court may make such a correction on motion by a party or on its own initiative, with or without notice.

However, once an appeal from the judgment or order has been docketed in the appellate court, such a correction can be made only with leave of the appellate court.

61
Q

post trial: Relief of a Judgment

A

Rule 60(b) allows a court to relieve a party from a final judgment or order for a motion filed:

(1) Within a reasonable time, and no later than one year following the entry of the judgment or order for:
- Mistake, inadvertence, surprise, or excusable neglect;
- Newly discovered evidence that could not have been earlier discovered with reasonable diligence; or
- Fraud (intrinsic or extrinsic), misrepresentation, or misconduct (failing to disclose documents during discovery) by an opposing party; or

(2) Within a reasonable time, with no definite limiting period, on the grounds that:
* a judgment is void (like there was never personal jurisdiction);
* a judgment has been satisfied, released, or discharged;
* a judgment was based on a judgment that was reversed or vacated; a
* pplying the judgment prospectively is no longer equitable;
* or for any other reason that justifies relief.
- add as more notes: The grounds of mistake, inadvertence, surprise, or excusable neglect include a judge’s error of law, even if the error is not an obvious legal mistake.

In sum, Rule 60(b) allows a court to exercise its equitable jurisdiction to relieve a party from a judgment in any case in which enforcing the judgment would work injustice, and the party seeking relief or its counsel was not guilty of misconduct or gross negligence. If a motion for relief is made, but the court lacks the authority to rule because of the pendency of an appeal, then the court may (i) defer, (ii) deny the motion, or (iii) state that it would grant the motion upon remand. Rule62.1.
- The court may provide any relief that it believes to be appropriate based on the evidence, and it is not limited to the relief requested in the pleadings.

62
Q

post trial: Alteration of judgment

A

party may make a motion for the court to alter or amend a judgment under Rule 59(e), this motion must be made within 28 days of the entry of the judgment.

This motion can be made on many grounds, including an intervening change in controlling law

63
Q

post trial: motion to set aside a judgment for fraud

A

a motion to set aside a judgment for fraud on the court is available only for fraud that seriously affects the integrity of the normal process of adjudication, such as bribery of a judge or a juror or the fabrication of evidence by a party in which an attorney is implicated.

Mere perjury by a witness does not itself constitute a fraud on the court.

64
Q

FL supreme court appellate jurisdiction

A

The Florida Supreme Court has discretionary (not mandatory) appellate jurisdiction over any order or judgment of a trial court certified by the district court of appeal in which an appeal is pending to be of great public importance or to have a great effect on the proper administration of justice throughout the state.

65
Q

discretionary standard of review

A

many decisions by trial ct judges are discretionary (admissibility of evidence, sanctions for violating discovery rules, granting/denial motion to transfer venue/sever actions)